Is T a One-to-One Linear Operator?

Click For Summary
To prove that the linear operator T is one-to-one, it is established that if ||T(x)|| = ||x|| for all x in the finite-dimensional vector space V, then T(x) = T(y) implies x = y. The discussion suggests using a proof by contradiction, assuming T(x) = T(y) for some x and y in V where x ≠ y. This leads to the conclusion that ||T(x)|| = ||T(y)||, which implies <x,x> = <y,y>, but does not directly show x = y. An alternative approach is proposed, focusing on proving that the kernel of T is trivial, i.e., ker(T) = {0}.
H12504106
Messages
4
Reaction score
0

Homework Statement



Let T be a linear operator on a finite dimensional vector space V. Suppose ||T(x)|| = ||x|| for all x in V, prove that T is one to one.

Homework Equations



||T(x)||^2 = <T(x),T(x)>
||x||^2 = <x,x>

The Attempt at a Solution



Suppose T(x) = T(y) x, y in V
Then ||T(x)|| = ||T(y)||
and so <T(x),T(x)> = <T(y),T(y)>
By assumption, we have <x,x> = <y,y>
But then i can't proceed on to show that x=y
 
Physics news on Phys.org
H12504106 said:

Homework Statement



Let T be a linear operator on a finite dimensional vector space V. Suppose ||T(x)|| = ||x|| for all x in V, prove that T is one to one.

Homework Equations



||T(x)||^2 = <T(x),T(x)>
||x||^2 = <x,x>

The Attempt at a Solution



Suppose T(x) = T(y) x, y in V
Then ||T(x)|| = ||T(y)||
and so <T(x),T(x)> = <T(y),T(y)>
By assumption, we have <x,x> = <y,y>
But then i can't proceed on to show that x=y
One form of the definition for one-to-one is this:
For all x and y in V, T(x) = T(y) ==> x = y.
You can do a proof by contradiction by assuming that for some x and y in V, T(x) = T(y) and x \neq y.

You need to use the fact that T is a linear operator.
 
If you've seen the right theorem, proving that \ker(T) = \{0\} might be a slightly easier way to do this.
 
Question: A clock's minute hand has length 4 and its hour hand has length 3. What is the distance between the tips at the moment when it is increasing most rapidly?(Putnam Exam Question) Answer: Making assumption that both the hands moves at constant angular velocities, the answer is ## \sqrt{7} .## But don't you think this assumption is somewhat doubtful and wrong?

Similar threads

  • · Replies 11 ·
Replies
11
Views
2K
  • · Replies 24 ·
Replies
24
Views
4K
  • · Replies 3 ·
Replies
3
Views
2K
  • · Replies 43 ·
2
Replies
43
Views
4K
Replies
3
Views
2K
  • · Replies 1 ·
Replies
1
Views
2K
  • · Replies 1 ·
Replies
1
Views
2K
  • · Replies 3 ·
Replies
3
Views
2K
Replies
12
Views
2K
Replies
8
Views
2K